Problem with Limits using L'Hospital's Rule

  • Thread starter CharlesL
  • Start date
  • Tags
    Limits
In summary, the limit of the equation lim \psi\rightarrow1 of \psi^(1/(\psi-1)) is e. Two possible solutions were discussed, and both resulted in the same answer of e.
  • #1
CharlesL
17
0

Homework Statement


Determine the limit of

lim [tex]\psi[/tex][tex]\rightarrow[/tex]1 of [tex]\psi[/tex]^(1/([tex]\psi[/tex]-1))


Homework Equations


psigraph.jpg



The Attempt at a Solution



Let y = [tex]\psi[/tex]^(1/([tex]\psi[/tex]-1))
ln y = ln [tex]\psi[/tex]^(1/([tex]\psi[/tex]-1))

lim [tex]\psi[/tex][tex]\rightarrow[/tex]1 ln y = lim [tex]\psi[/tex][tex]\rightarrow[/tex]1 of (1/([tex]\psi[/tex]-1)) (ln [tex]\psi[/tex])

Differentiate

lim [tex]\psi[/tex][tex]\rightarrow[/tex]1 ln y = -1/([tex]\psi[/tex]-1)2 x (1/[tex]\psi[/tex])

lim [tex]\psi[/tex][tex]\rightarrow[/tex]1 ln y = 2/([tex]\psi[/tex]3+3[tex]\psi[/tex]2+3[tex]\psi[/tex]+1)

ln y =1/4
y = e1/4

Does e1/4 = e?
 
Physics news on Phys.org
  • #2
L'Hopital's rule works for f(x)/g(x) and then you get f'(x)/g'(x). Try rewriting the step before you differentiate as a fraction and not a product.
 
  • #3
Thank you for your reply.

I wonder which is the correct solution

solution (a)

ln y = 1/([tex]\psi[/tex]-1) x ln [tex]\psi[/tex]

ln y = ln [tex]\psi[/tex] x ([tex]\psi[/tex]-1)

ln y = 1/[tex]\psi[/tex]

ln y = 1/1

y = e1

or solution (b)

ln y = 1/([tex]\psi[/tex]-1) x ln [tex]\psi[/tex]

ln y = ln [tex]\psi[/tex] / ([tex]\psi[/tex]-1)

ln y = 1/[tex]\psi[/tex]

ln y = 1/1

y = e1
 
Last edited:
  • #4
I did it the second way, assuming you just didn't feel like typing out that you were still dealing with limits
 
  • #5
Thank you Hogger for your point outs. Appreciate it. Have a nice day



Charles
 

1. What is L'Hospital's Rule?

L'Hospital's Rule is a mathematical theorem that provides a method for evaluating limits in cases where the limit of a function approaches either 0 or infinity. It states that the limit of the quotient of two functions with the same limit at a certain point can be found by taking the limit of the quotient of their derivatives at that point.

2. When should L'Hospital's Rule be used?

L'Hospital's Rule should be used when evaluating limits that result in indeterminate forms, such as 0/0 or infinity/infinity. It can also be used when evaluating limits at infinity or negative infinity.

3. What are the limitations of L'Hospital's Rule?

L'Hospital's Rule has several limitations, including that it only applies to limits of the form 0/0 or infinity/infinity. It also may not work for limits involving trigonometric functions or logarithmic functions.

4. How do I know when to apply L'Hospital's Rule?

If you encounter an indeterminate form (such as 0/0 or infinity/infinity) when evaluating a limit, it is a good indication that L'Hospital's Rule may be applicable. You can also check if the limit is at infinity or negative infinity, as this is another case where the rule can be used.

5. Can L'Hospital's Rule be used for one-sided limits?

Yes, L'Hospital's Rule can be used for both one-sided limits (limits approaching from only one direction) and two-sided limits (limits approaching from both directions). However, you may need to apply the rule separately for each one-sided limit.

Similar threads

  • Calculus and Beyond Homework Help
Replies
14
Views
245
  • Calculus and Beyond Homework Help
Replies
5
Views
286
  • Calculus and Beyond Homework Help
Replies
3
Views
1K
  • Calculus and Beyond Homework Help
Replies
8
Views
1K
  • Calculus and Beyond Homework Help
Replies
8
Views
1K
  • Calculus and Beyond Homework Help
Replies
14
Views
1K
  • Calculus and Beyond Homework Help
Replies
3
Views
909
  • Calculus and Beyond Homework Help
Replies
1
Views
1K
  • Calculus and Beyond Homework Help
Replies
1
Views
1K
  • Calculus and Beyond Homework Help
Replies
4
Views
1K
Back
Top